Những câu hỏi liên quan
Nguyễn Tấn Dũng
Xem chi tiết
Gallavich
Xem chi tiết
Eren
Xem chi tiết
Akai Haruma
29 tháng 9 2017 lúc 0:50

Lời giải:

Áp dụng hệ quả của BĐT AM-GM:

\(\text{VT}^2=\left[\frac{1}{a(a+1)}+\frac{1}{b(b+1)}+\frac{1}{c(c+1)}\right]^2\geq 3\left(\frac{1}{ab(a+1)(b+1)}+\frac{1}{bc(b+1)(c+1)}+\frac{1}{ca(a+1)(c+1)}\right)\)

\(\Leftrightarrow \text{VT}^2\geq 3.\frac{a^2+b^2+c^2+a+b+c}{abc(a+1)(b+1)(c+1)}\geq 3.\frac{a+b+c+ab+bc+ac}{abc(a+1)(b+1)(c+1)}\)

\(\Leftrightarrow \text{VT}^2\geq \frac{3}{abc}-\frac{3(abc+1)}{abc(a+1)(b+1)(c+1)}\) \((1)\)

Ta sẽ cm \((a+1)(b+1)(c+1)\geq (1+\sqrt[3]{abc})^3\). Thật vậy:

Áp dụng BĐT AM-GM:

\(\frac{a}{a+1}+\frac{b}{b+1}+\frac{c}{c+1}\geq 3\sqrt[3]{\frac{abc}{(a+1)(b+1)(c+1)}}\)

\(\frac{1}{a+1}+\frac{1}{b+1}+\frac{1}{c+1}\geq 3\sqrt[3]{\frac{1}{(a+1)(b+1)(c+1)}}\)

Cộng theo vế: \(\Rightarrow 3\geq \frac{3(\sqrt[3]{abc}+1)}{\sqrt[3]{(a+1)(b+1)(c+1)}}\)

\(\Rightarrow (a+1)(b+1)(c+1)\geq (\sqrt[3]{abc}+1)^3\) (2)

Từ \((1),(2)\Rightarrow \text{VT}^2\geq \frac{3}{abc}-\frac{3(abc+1)}{abc(1+\sqrt[3]{abc})^3}=\frac{9}{\sqrt[3]{a^2b^2c^2}(1+\sqrt[3]{abc})^2}=\text{VP}^2\)

\(\Leftrightarrow \text{VT}\geq \text{VP}\) (đpcm)

Dấu bằng xảy ra khi \(a=b=c=1\)

Bình luận (0)
Nguyễn Huy Thắng
28 tháng 9 2017 lúc 1:45

ap dung bdt holder

Bình luận (1)
Thảo Vi
Xem chi tiết
Akai Haruma
8 tháng 3 2021 lúc 21:46

Bài 1:

Áp dụng BĐT AM-GM ta có:

$\frac{1}{a+1}+\frac{1}{b+1}+\frac{1}{c+1}\geq 3\sqrt[3]{\frac{1}{(a+1)(b+1)(c+1)}}$

$\frac{a}{a+1}+\frac{b}{b+1}+\frac{c}{c+1}\geq 3\sqrt[3]{\frac{abc}{(a+1)(b+1)(c+1)}}$

Cộng theo vế và thu gọn:

$\frac{a+1}{a+1}+\frac{b+1}{b+1}+\frac{c+1}{c+1}\geq \frac{3(1+\sqrt[3]{abc})}{\sqrt[3]{(a+1)(b+1)(c+1)}}$

$\Leftrightarrow 3\geq \frac{3(1+\sqrt[3]{abc})}{\sqrt[3]{(a+1)(b+1)(c+1)}}$

$\Rightarrow (a+1)(b+1)(c+1)\geq (1+\sqrt[3]{abc})^3$

Ta có đpcm.

Bình luận (0)
Akai Haruma
8 tháng 3 2021 lúc 21:49

Bài 2:

$a^3+a^3+a^3+a^3+b^3+c^3\geq 6\sqrt[6]{a^{12}b^3c^3}=6a^2\sqrt{bc}$

$b^3+b^3+b^3+b^3+a^3+c^3\geq 6b^2\sqrt{ac}$

$c^3+c^3+c^3+c^3+a^3+b^3\geq 6c^2\sqrt{ab}$

Cộng theo vế và rút gọn thu được:

$a^3+b^3+c^3\geq a^2\sqrt{bc}+b^2\sqrt{ac}+c^2\sqrt{ab}$ 

Ta có đpcm.

Dấu "=" xảy ra khi $a=b=c$

Bình luận (0)
Akai Haruma
8 tháng 3 2021 lúc 21:50

Bài 3:

Áp dụng BĐT Cauchy-Schwarz:

$\frac{a^2}{b+c}+\frac{b^2}{c+a}+\frac{c^2}{a+b}\geq \frac{(a+b+c)^2}{b+c+c+a+a+b}=\frac{(a+b+c)^2}{2(a+b+c)}=\frac{a+b+c}{2}$

Ta có đpcm

Dấu "=" xảy ra khi $a=b=c$

Bình luận (0)
Lưu Thị Thảo Ly
Xem chi tiết
Akai Haruma
23 tháng 3 2017 lúc 1:08

Lời giải:

Áp dụng BĐT Cauchy:

\(2\sqrt{a(3a+b)}=\sqrt{4a(3a+b)}\leq \frac{4a+3a+b}{2}\)

Tương tự \(2\sqrt{b(3b+a)}\leq \frac{4b+3b+a}{2}\)

\(\Rightarrow 2(\sqrt{a(3a+b)}+\sqrt{b(3b+a)})\leq \frac{8a+8b}{2}=4(a+b)\)

\(\Rightarrow \sqrt{a(3a+b)}+\sqrt{b(3b+a)}\leq 2(a+b)\)

\(\Rightarrow \frac{a+b}{\sqrt{a(3a+b)}+\sqrt{b(3b+a)}}\geq \frac{a+b}{2(a+b)}=\frac{1}{2}\) (đpcm)

Dấu bằng xảy ra khi \(a=b>0\)

Bình luận (0)
Phạm Mỹ Châu
Xem chi tiết
Lightning Farron
26 tháng 5 2018 lúc 18:24

Từ \(a^2+b^2+c^2=3\Rightarrow a+b+c\le3\)

Ta có: \(\sqrt{\dfrac{9}{\left(a+b\right)^2}+c^2}+\sqrt{\dfrac{9}{\left(b+c\right)^2}+a^2}+\sqrt{\dfrac{9}{\left(c+a\right)^2}+b^2}\)

\(\ge\sqrt{9\left(\dfrac{1}{a+b}+\dfrac{1}{b+c}+\dfrac{1}{c+a}\right)^2+\left(a+b+c\right)^2}\)

\(\ge\sqrt{9\cdot\left(\dfrac{9}{2\left(a+b+c\right)}\right)^2+\left(a+b+c\right)^2}\)

Cần chứng minh \(\sqrt{9\cdot\left(\dfrac{9}{2\left(a+b+c\right)}\right)^2+\left(a+b+c\right)^2}\ge\dfrac{3\sqrt{13}}{2}\)

\(\Leftrightarrow9\left(\dfrac{9}{2t}\right)^2+t^2\ge\dfrac{117}{4}\left(t=a+b+c\le3\right)\)

\(\Leftrightarrow\dfrac{\left(t-3\right)\left(2t-9\right)\left(t+3\right)\left(2t+9\right)}{4t^2}\ge0\)*Đúng*

Bình luận (1)
Thanh Mai Đinh
Xem chi tiết
Nguyễn Lê Phước Thịnh
9 tháng 7 2022 lúc 14:10

Bài 1: 

a: \(A=\left(\sqrt{x}+\sqrt{y}-\dfrac{\left(\sqrt{x}-\sqrt{y}\right)\left(x+\sqrt{xy}+y\right)}{\left(\sqrt{x}-\sqrt{y}\right)\left(\sqrt{x}+\sqrt{y}\right)}\right)\cdot\dfrac{\sqrt{x}+\sqrt{y}}{x-\sqrt{xy}+y}\)

\(=\dfrac{x+2\sqrt{xy}+y-x-\sqrt{xy}-y}{\sqrt{x}+\sqrt{y}}\cdot\dfrac{\sqrt{x}+\sqrt{y}}{x-\sqrt{xy}+y}\)

\(=\dfrac{\sqrt{xy}}{x-\sqrt{xy}+y}\)

b: \(\sqrt{xy}>=0;x-\sqrt{xy}+y>0\)

Do đó: A>=0

Bình luận (0)
Lizy
Xem chi tiết
Nguyễn Việt Lâm
17 tháng 1 lúc 20:30

\(A=\dfrac{2}{\sqrt{ab}}:\left(\dfrac{\sqrt{a}-\sqrt{b}}{\sqrt{ab}}\right)^2-\dfrac{a+b}{\left(\sqrt{a}-b\right)^2}\)

\(=\dfrac{2}{\sqrt{ab}}.\dfrac{ab}{\left(\sqrt{a}-\sqrt{b}\right)^2}-\dfrac{a+b}{\left(\sqrt{a}-\sqrt{b}\right)^2}\)

\(=\dfrac{2\sqrt{ab}}{\left(\sqrt{a}-\sqrt{b}\right)^2}-\dfrac{a+b}{\left(\sqrt{a}-\sqrt{b}\right)^2}\)

\(==\dfrac{-\left(a-2\sqrt{ab}+b\right)}{\left(\sqrt{a}-\sqrt{b}\right)^2}=\dfrac{-\left(\sqrt{a}-\sqrt{b}\right)^2}{\left(\sqrt{a}-\sqrt{b}\right)^2}=-1\)

Bình luận (0)
missing you =
Xem chi tiết
Nguyễn Việt Lâm
28 tháng 6 2021 lúc 10:06

Đề bài hình như bị sai em, thay điểm rơi ko thỏa mãn

Biểu thức là \(a+b+\sqrt{2\left(a+c\right)}\) mới đúng

Bình luận (2)
missing you =
28 tháng 6 2021 lúc 10:11

em cũng nghĩ thế mới dùng đc BDT AM-GM 3 số đúng ko thầy :)

Bình luận (1)